UPSC PRELIMINARY QUESTIONS PAPER – I - 2017 WITH ANSWERS SET D

1. Which one of the following was a very helps to ensure that poor developing important seaport in the Kakatiya kingdom? countries most vulnerable to climate (a) Kakinada change increase their capacities to adapt (b) Motupalli to the effects of climate change, in (c) Machilipatnam (Masulipatnam) support of the achievement of the (d) Nelluru Millenium Development Goals (MDGs). Explanation: Where this benefits their poverty reduction Marco Polo referred to the kingdom as objectives, the Alliance also helps such Mutfili, which was the name for the area countries to participate in the global around a major port of the Kakatiya climate change mitigation effort. dynasty, now known as Masulipatnam. As such, it is becoming clear that the option 2 is correct. 2. With reference to 'Global Climate Change Source: Alliance', which of the following statements http://www.gcca.eu/about-the-gcca/what- is/are correct? is-the-gcca 1) It is an initiative of the European Union. 2) It provides technical and financial 3. With reference to the religious history of, support to targeted developing India, consider the following statements: countries to integrate climate change 1) Sautrantika and Sammitiya were the into their development policies and sects of Jainism. 2) Sarvastivadinleld that the 'constituents budgets. of phenomena were not wholly 3) It is coordinated by World Resources momentary, but existed forever in a Institute (WRI) and World Business latent form. Council for Sustainable Development Which of the statements given above is/are (WBCSD). correct? Select the correct answer using the codes (a) 1 only given below: (b) 2 only (a) 1 and 2 only (c) Both 1 and 2 (b) 3 only (d) Neither 1 nor 2 (c) 2 and 3 only Explanation: (d) 1, 2 and 3 Statement 1 is wrong Explanation:  Sautrāntika & Saṃmitīya are both The GCCA was established by the European schools of Buddhism, Union (EU) in 2007 to strengthen dialogue Statement 2 is correct and cooperation with developing countries,  The word Sarvastivadin is derived in particular least developed countries from the Sanskrit terms 'sarvam', 'all (LDCs) and small island developing States things' and 'asti', 'exist' (SIDS).  The word Sarvastivadin in Sanskrit By fostering effective dialogue and denotes 'those who believe that all cooperation on climate change, the Alliance things exist'

UPSC 2017 Preliminary Exam Q&A Video Discussion with Smart Leaders IAS Page 1 of 39 will be available in our youtube channel “Smart Leaders IAS Online” at 9:00pm

4. Mediterranean Sea is a border of which of 5. With reference to 'National Investment and the following countries? Infrastructure Fund', which of the following 1) Jordan statements is/are correct? 2) Iraq 1) It is an organ of NITI Aayog. 3) Lebanon 2) It has a corpus of 4,00,000 crore at 4) Syria present. Select the correct answer using the codes Select the correct answer using the code given below: given below: (a) 1, 2 and 3 only (a) 1 only (b) 2 and 3 only (b) 2 only (c) 3 and 4 only (c) Both 1 and 2 (d) 1, 3 and 4 only (d) Neither 1 nor 2 Explanation: Explanation: Its creation was announced in the Union Budget 2015-16. NIIF got registered with SEBI as Category II Alternative Investment Fund (AIF) on December 28, 2015 under the SEBI (Alternative Investments Funds) Regulations, 2012 ("AIF Regulations"). The corpus of the Fund is proposed to be of the order of Rs.40,000 cr (US$6 billion) wherein the Government of India would invest 49%, circa Rs.20,000 cr (US$3 billion). NIIF is expected play the role of a catalyst for supporting investments in infrastructure with the objective of maximizing economic impact through The countries with coastlines on the investments in commercially viable Mediterranean Sea are Albania, Algeria, projects, both greenfield and brownfield, Bosnia and including stalled projects. NIIF shall also Herzegovina, Croatia, Cyprus, Egypt, Franc consider other nationally important e, Greece, Israel, Italy, Lebanon, Libya, Ma projects, if found commercially viable. lta, Morocco, Monaco, Montenegro, Sloven ia, Spain, Syria, Tunisiaand Turkey. In NIIF will raise third party capital for the addition, the Gaza Strip and the British remaining Rs.20,000 crore (US$ 3 billion), Overseas Territories of Gibraltar from long term international investors, and Akrotiri and Dhekelia have coastlines such as a Sovereign Wealth Funds, on the sea. Insurance and Pension Funds, endowments Source: etc. National Investment and Infrastructure Fund (NIIF) is formed as a trust (set up https://en.wikipedia.org/wiki/Mediterrane an_Sea under the provisions of Indian Trusts Act 1882) created by the Government of India. Source: http://niifindia.com/home-page.html UPSC 2017 Preliminary Exam Q&A Video Discussion with Smart Leaders IAS Page 2 of 39 will be available in our youtube channel “Smart Leaders IAS Online” at 9:00pm

6. The Global Infrastructure Facility is a/an delivering on objectives and principles. The (a) ASEAN initiative to upgrade Governing Council comprises infrastructure in Asia and financed by representatives of funding and technical credit from the Asian Development Bank. partners and representatives of emerging (b) World Bank collaboration that markets and developing economies, and is facilitates the preparation and co-chaired by the World Bank Group and a Funding Partner. structuring of complex infrastructure Source: Public-Private Partnerships (PPPs) to http://www.globalinfrafacility.org/what-is- enable mobilization of private sector the-gif and institutional investor capital. (c) Collaboration among the major banks of 7. For election to the Lok Sabha, a nomination the world working with the OECD and paper can be filed by focused on expanding the set of (a) anyone residing in India. infrastructure projects that have the (b) a resident of the constituency from which potential to mobilize private investment. the election is to be contested. (d) UNCTAD funded initiative that seeks to (c) any citizen of India whose name finance and facilitate infrastructure appears in the electoral roll of a development in the world. constituency. (d) any citizen of India. Explanation: Explanation: The Global Infrastructure Facility (GIF) is a For contesting an election as a candidate a person must be registered as a voter. Sec 4 partnership among governments, (d) of Representation People Act, 1951 multilateral development banks, private precludes a person from contesting unless sector investors, and financiers. It is he is an elector in any parliamentary designed to provide a new way to constituency. Section 5 (c) of R. P. Act, 1951 collaborate on preparing, structuring, and has a similar provision for Assembly implementing complex projects that no Constituencies. single institution could handle on its own. Source: The comprehensive project support http://eci.nic.in/eci_main1/Contesting.aspx provided by the GIF draws on the combined expertise of its technical and advisory 8. Consider the following statements: partners. This group, which includes 1) In India, the Himalayas are spread over commercial banks and institutional five States only. investors, ensures that well-structured and 2) Western Ghats are spread over five bankable infrastructure projects are brought to market in a way that sustainably States only. meet the needs of governments and service 3) Pulicat Lake is spread over two States users. Funding partners provide financial only. contributions to the GIF. The GIF Which of the statements given above is/are partnership is overseen by a Governing correct? Council that supervises strategic (a) 1 and 2 only programming and funds management as (b) 3 only well as the development of operational (c) 2 and 3 only policies and procedures. It also holds the (d) 1 and 3 only GIF’s management accountable for UPSC 2017 Preliminary Exam Q&A Video Discussion with Smart Leaders IAS Page 3 of 39 will be available in our youtube channel “Smart Leaders IAS Online” at 9:00pm

Explanation: 10. With reference to the role of UN-Habitat in Statement 1 is wrong the United Nations programme working  The Indian Himalayan Region is the towards a better urban future, which of the section of the Himalayas within India, statements is/are correct ? spanning the states of Jammu & 1) UN-Habitat has been mandated by the Kashmir, Himachal Pradesh, United Nations General Assembly to Uttarakhand, Sikkim, Arunachal promote socially and environmentally Pradesh, as well as the hill regions of sustainable towns and cities to provide two states - Assam and West Bengal. adequate shelter for all. Statement 2 is wrong 2) Its partners are either governments or  Western Ghats are spread in six states local urban authorities only. viz. Gujarat, Maharashtra, Goa, 3) UN-Habitat contributes to the overall Karnataka, Tamil Nadu, Kerala and two objective of the United Nations system Union Territories viz. Dadra & Nagar to reduce poverty and to promote Haveli and Pondicherry access to safe drinking water and basic Statement 3 is correct sanitation.  Pulicat Lake formerly Pralaya Kaveri is Select the correct answer using the codes the second largest brackish water lake given below: or lagoon in India, after Chilika Lake. (a) 1, 2 and 3  It straddles the border of Andhra (b) 1 and 3 only Pradesh and Tamil Nadu states with (c) 2 and 3 only over 40% of it in Andhra Pradesh and (d) 1 only 60% in Tamil Nadu Explanation: UN-Habitat is the United Nations 9. Biological Oxygen Demand (BOD) is a programme working towards a better standard criterion for urban future. Its mission is to promote (a) Measuring oxygen levels in blood socially and environmentally sustainable (b) Computing oxygen levels in forest human settlements development and the ecosystems achievement of adequate shelter for all. (c) Pollution assay in aquatic ecosystems In many places around the world, the (d) Assessing oxygen levels in high altitude effects can already be felt: lack of proper regions housing and growth of slums, inadequate Explanation: and out-dated infrastructure – be it roads, Biochemical oxygen demand or BOD is a public transport, water, sanitation, or chemical procedure for determining the electricity – escalating poverty and amount of dissolved oxygen needed by unemployment, safety and crime problems, aerobic biological organisms in a body of pollution and health issues, as well as water. It is not a precise quantitative test, poorly managed natural or man-made although it is widely used as an indication disasters and other catastrophes due to the of the organic quality of water. effects of climate change. The consequences of high BOD are the same Through drawing together cooperation as those for low dissolved oxygen: aquatic with committed partners, relevant organisms become stressed, suffocate, and stakeholders, and urban actors, including at die. all levels of government as well as the

UPSC 2017 Preliminary Exam Q&A Video Discussion with Smart Leaders IAS Page 4 of 39 will be available in our youtube channel “Smart Leaders IAS Online” at 9:00pm

private sector, UN-Habitat is applying its Explanation: technical expertise, normative work and Under NSQF, the learner can acquire the capacity development to implement the certification for competency needed at any New Urban Agenda and Sustainable level through formal, non-formal or Development Goal 11 – to make cities informal learning. inclusive, safe, resilient and sustainable. Mandated by the UN General Assembly in Specific outcomes expected from 1978 to address the issues of urban growth, implementation of NSQF are: it is a knowledgeable institution on urban Mobility between vocational and general development processes, and understands education by alignment of degrees with the aspirations of cities and their residents. NSQF For forty years, UN-Habitat has been Source: working in human settlements throughout http://www.skilldevelopment.gov.in/nsqf.h the world, focusing on building a brighter tml future for villages, towns, and cities of all sizes. Because of these four decades of extensive experience, from the highest 12. In the context of Indian history, the levels of policy to a range of specific principle of `Dyarchy (diarchy)' refers to technical issues, UN-Habitat has gained a (a) Division of the central legislature into unique and a universally acknowledged two houses. expertise in all things urban. (b) Introduction of double government i.e., Source: Central and State governments. https://unhabitat.org/about-us/un-habitat- (c) Having two sets of rulers; one in London at-a-glance/# and another in Delhi. 11. With reference to 'National (d) Division of the subjects delegated to Skills Qualification Framework (NSQF)', the provinces into two categories. which of the statements given below is/are Explanation: correct?  The principle of dyarchy was a division of the executive branch of each 1) Under NSQF, a learner can acquire the provincial government into certification for competency only authoritarian and popularly responsible through formal learning. sections. 2) An outcome expected from the  The Act provided a dual form of implementation of NSQF is the mobility government (a "diarchy") for the major between vocational and general provinces education.  In each such province, control of some Select the correct answer using the code areas of government, the "transferred list", were given to a Government of given below: ministers answerable to the Provincial (a) 1 only Council (b) 2 only (c) Both 1 and 2 (d) Neither 1 nor 2

UPSC 2017 Preliminary Exam Q&A Video Discussion with Smart Leaders IAS Page 5 of 39 will be available in our youtube channel “Smart Leaders IAS Online” at 9:00pm

13. Consider the following in respect of digital infrastructure and generate 'National Career Service' : employment opportunities for all. 1) National Career Service is an initiative The Career Centres will serve as a platform of the Department of Personnel and for addressing all career related needs of Training, Government of India. the youth and students seeking counselling, 2) National Career Service has been job search assistance services and launched in a Mission Mode to improve employment the employment opportunities to Source: uneducated youth of the country. https://en.wikipedia.org/wiki/National_Ca Which of the above statements is/are reer_Service_(India) correct? (a) 1 only 14. Which of the following statements best (b) 2 only describes the term 'Scheme for Sustainable (c) Both 1 and 2 Structuring of Stressed Assets (S4A)', (d) Neither 1 nor 2 recently seen in the news ? Explanation: (a) 'It is a procedure for considering National Career Service (NCS) project[1] is ecological costs of developmental an initiative launched by the Ministry of schemes formulated by the Government. Labour and Employment (b) It is a scheme of RBI for reworking the (India)Government of India as a Mission financial structure of big corporate Mode Project for establishing quick and entities facing genuine difficulties. efficient career related services. It was (c) It is a disinvestment plan of the launched by Prime Minister Narendra Government regarding Central Public Modi on 20 July 2015 as part of Sector Undertakings. government’s focus on providing right skills (d) It is an important provision in 'The and generating employment.[2] Insolvency and Bankruptcy Code' recently The key success factors of NCS initiative implemented by the Government. revolve around strengthening the existing Explanation: employment exchange ecosystem with an The scheme was launched by RBI in June ICT enabled platform and reaching out to 2016 to deal with the large stressed assets masses through multiple delivery channels of the corporate sector with banks. and partnerships with existing job market The debt will be bifurcated into Sustainable players. Caught in a vicious cycle of and Unsustainable loans evaluated by an decreasing job seeker registrations and independent agency hired by the lending vacancy notifications by employers, these bank. exchanges needed a transformation for Source: better delivery of employment https://www.rbi.org.in/scripts/BS_PressRe services.[3] Employment exchanges have leaseDisplay.aspx?prid=37210 been transformed to focus more on http://www.thehindubusinessline.com/opi counselling related needs of candidates nion/columns/slate/all-you-wanted-to- with establishment of career centers[4] know-about-s4a/article9288558.ece NCS is the converging point for Government of India three major initiatives, Skill India, Digital India and Make in India, that aim to provide skilled manpower, improve UPSC 2017 Preliminary Exam Q&A Video Discussion with Smart Leaders IAS Page 6 of 39 will be available in our youtube channel “Smart Leaders IAS Online” at 9:00pm

15. Consider the following statements: 2) An IOD phenomenon can influence an El 1) Climate and Clean Air Coalition (CCAC) Nino's' impact on the monsoon. to Reduce Short Lived Climate Select the correct answer using the codes Pollutants is a unique initiative of G20 given below: group of countries. (a) 1 only 2) The CCAC focuses on methane, black (b) 2 only carbon and hydrofluorocarbons. (c) Both 1 and 2 Which of the statements given above is/are (d) Neither 1 nor 2 correct? Explanation: (a) 1 only Statement 1 is correct (b) 2 only  The Indian Ocean Dipole (IOD), also (c) Both 1 and 2 known as the Indian Niño, is an (d) Neither 1 nor 2 irregular oscillation of sea-surface Explanation: temperatures in which the western The Climate and Clean Air Coalition formed Indian Ocean becomes alternately in 2012 is the only global effort that unites warmer and then colder than the governments, civil society and private eastern part of the ocean sector, committed to improving air quality Statement 2 is correct and protecting the climate in next few  Indian monsoon depends upon not only decades by reducing short-lived climate El Nino La Nina but also IOD and other pollutants across sectors. such ocean phenomena. As mentioned United Nations Environment Programme above, an IOD can either aggravate or (UNEP) hosts CCAC Secretariat weaken the impact of El Nino on Indian Short-lived climate pollutants (SLCPs) are monsoon. If there is a positive IOD, it agents that have a relatively short lifetime can bring good rains to India despite of in the atmosphere - a few days to a few an El Nino year. decades - and a warming influence on climate. The main short lived climate 17. If you want to see gharials in their natural pollutants are black carbon, methane and habitat, which one of the following is the tropospheric ozone. best place to visit? The option (1) is wrong while the option (a) Bhitarkanika Mangroves (2) is correct. (b) Chambal River Source: (c) Pulicat Lake http://www.ccacoalition.org/en/science- (d) Deepor Beel resources Explanation: Gharials, sometimes called gavials, are a 16. With reference to 'Indian Ocean Dipole type of Asian crocodilian distinguished by (IOD)' sometimes mentioned in the news their long, thin snouts. Crocodilians are a while forecasting Indian monsoon, which of group that includes crocodiles, alligators, the following statements is/are correct? caimans, and more. 1) IOD phenomenon is characterized by a Once found from Pakistan to Myanmar difference in sea surface temperature (Burma), the reptile's range has shrunk to between tropical Western Indian Ocean two countries: India, along the Chambal, and tropical Eastern Pacific Ocean.

UPSC 2017 Preliminary Exam Q&A Video Discussion with Smart Leaders IAS Page 7 of 39 will be available in our youtube channel “Smart Leaders IAS Online” at 9:00pm

Girwa, and Son Rivers; and Nepal, along the http://ions.gov.in/about_ions Narayani River. The gharial reserves of India are located in 19. The painting of Bodhisattva Padmapani three States – Uttar Pradesh, Madhya is one of the most famous and oft- Pradesh and Rajasthan. illustrated paintings at Source: (a) Ajanta http://www.nationalgeographic.com/anim (b) Badami

als/reptiles/g/gharial/ (c) Bagh (d) Ellora

Explanation: 18. Consider the following in respect of Indian  The most famous painting in Ajanta is Ocean Naval Symposium (IONS) : the Bodhisattva Padmapani (Padmapani 1) Inaugural IONS was held in India in in Sanskrit literally translates into "one 2015 under the chairmanship of the who holds the lotus"). Indian Navy.  Paintings at Badami are housed in 2) IONS is a voluntary initiative that seeks rock-cut cave temples, mostly belong to to increase maritime co-operation 6th and 8th centuries AD among navies of the littoral states of the  The caves at Vatapi were fully decorated Indian Ocean Region. with murals; many of them were Which of the above statements is/are inspired from . correct?  The Badami caves were a major source (a) 1 only of inspiration to the monuments at (b) 2 only Mahabalipuram

(c) Both 1 and 2  The Bagh Caves are a group of nine rock-cut monuments located in Madhya (d) Neither 1 nor 2 Pradesh Explanation:  The paintings on the wall and ceilings of The inaugural IONS-2008 was held in New the viharas of Bagh were executed in Delhi, India on 14 Feb 08. CNS, Indian Navy tempera was designated the Chairman IONS for the  These paintings are materialistic rather period 2008-10. A ‘Charter of than spiritualistic Business’ was mutually agreed to by the  Ellora Caves is one of the largest rock- Chiefs of the member-navies, which has cut monastery-temple caves complexes been forwarded to all the member navies in the world for ratification. The theme of the IONS-2008  The site presents monuments and was “Contemporary Trans-national artwork of Buddhism, and Challenges – International Maritime Jainism Connectivities”.  Cave 16 of Ellora The ‘Indian Ocean Naval Symposium’ features the (IONS) is a voluntary initiative that seeks to largest single increase maritime co-operation among monolithic rock navies of the littoral states of the Indian excavation in the Ocean Region by providing an open and world, the inclusive forum for discussion of regionally Kailasha temple, a relevant maritime issues. chariot shaped monument dedicated to Source:

UPSC 2017 Preliminary Exam Q&A Video Discussion with Smart Leaders IAS Page 8 of 39 will be available in our youtube channel “Smart Leaders IAS Online” at 9:00pm

20. Consider the following pairs: (a) 1 and 2 only Traditions Communities (b) 3 only 1) Chaliha Sahib - (c) 1 and 3 only Festival (d) 1, 2 and 3 2) Nanda Raj Jaat - Gonds Explanation: Yatra Improved water infiltration, less runoff and 3) Wari-Warkari - Santhals reduced evaporative losses in no-till Which of the above pairs is/are correctly systems can save from 5 to 12 inches per matched? year, making more water available for crop (a) 1 only production. Therefore, the statement (1) is (b) 2 and 3 only correct. (c) 1 and 3 only Throughout the year, residue protects the (d) None of the above soil from the sun and wind, reducing losses through evaporation. In winter it traps  Chalio / Chaliho also called moisture through winter snowfall. Crop Chaliho Sahib is a forty-day-long residue also reduces evaporation from the festival celebrated by Hindu Sindhi soil and keeps the soil surface cooler. It community reflects some sunlight energy back to the  This is a thanks-giving celebration in air and acts as insulation to keep the soil honor of and cooler in the heat of the summer. This for listening to their prayers cooler, moist surface allows better root  Nanda Raj Jat is a pilgrimage development during the growing season, and festival of Uttarakhand in India improving the standability of corn. This  The Jaat means Yatra or pilgrimage makes the statement (3) correct.  Pandharpur Wari or Wari (Vari) is While gypsum is mainly used to treat acidic an annual pilgrimage (yatra) to soils, as a calcium supplement and soil Pandharpur structure. Hence the statement (2) is  Santhals mainly celebrate the wrong. Karam festival which falls in the Source: month of September and October http://cropwatch.unl.edu/tillage/water  Gonds fair and festivals are http://cropwatch.unl.edu/conserving-soil- influenced from the Hindu and-water-no-till-and-crop-residue-unl- traditions. Keslapur Jathra is the important festival of the Gonds. In cropwatch-april-5-2013 this festival they worship the snake deity called Nagoba 22. Consider the following statements : The nation-wide 'Soil Health Card Scheme' 21. Which of the following practices can help in aims at 1) expanding the cultivable area under water conservation in agriculture? irrigation. 1) Reduced or zero tillage of the land 2) enabling the banks to assess the 2) Applying gypsum before irrigating the quantum of loans to be granted to field farmers on the basis of soil quality. 3) Allowing crop residue to remain in the 3) checking the overuse of fertilizers in field farmlands. Select the correct answer using the codes

given below:

UPSC 2017 Preliminary Exam Q&A Video Discussion with Smart Leaders IAS Page 9 of 39 will be available in our youtube channel “Smart Leaders IAS Online” at 9:00pm

Which of the above statements is/are Which of the pairs given above is/are correct? correctly matched? (a) 1 and 2 only (a) 1 only (b) 3 only (b) 2 and 3 only (c) 2 and 3 only (c) 1 and 3 only (d) 1, 2 and 3 (d) 1, 2 and 3 Explanation: Explanation: The measures to improve minimum Daily use consumables such as turmeric, support price and assistance like improved vermillion or cosmetics such as kajal and irrigation and rural electrification, the lipstick (both branded and unbranded), can incumbent NDA regime has laid emphasis contain high lead content. Brominated vegetable oil (BVO) is a on the Soil Health Card Scheme. complex mixture of plant-derived Launched by the central government in triglycerides that have been reacted to February 2015, the scheme is tailor-made contain atoms of the element bromine to issue ‘Soil card’ to farmers which will bonded to the molecules. Brominated carry crop-wise recommendations of vegetable oil is used primarily to help nutrients and fertilizers required for the emulsify citrus-flavored soft drinks, individual farms. This is aimed to help preventing them from separating during farmers to improve productivity through distribution. judicious use of inputs. Monosodium glutamate It was originally In the words of the union Agriculture synthesized by Japanese biochemist Kikunae Ikeda. He called this flavor umami. Minister Mohan Singh, this path- Source: breaking initiative would create a golden http://www.thehindu.com/news/cities/ba opportunity for the farmers to improve the ngalore/high-lead-content-in-everyday- productivity of their crops and also go for consumables/article7296361.ece diversification. This will certainly http://www.environmentalhealthnews.org contribute significantly to ensuring food /ehs/news/2011/brominated-battle-in- security of the country. sodas Source: http://www.foxnews.com/food- http://pib.nic.in/newsite/mbErel.aspx?reli drink/2014/11/12/foods-didnt-know- d=133855 contain-msg.html

24. Organic Light Emitting Diodes (OLEDs) are 23. Consider the following pairs: used to create digital display in many Commonly Unwanted or devices. create digital display in many used/ controversial devices. What are the advantages of OLED consumed chemicals likely to displays over Liquid Crystal displays ? materials be found in them 1) OLED displays can be fabricated on 1) Lipstick - Lead flexible plastic substrates. 2) Soft drinks - Brominated 2) Roll-up displays embedded in clothing vegetable oils can be made using OLEDs. 3) Chinese fast - Monosodium 3) Transparent displays are possible using food glutamate OLEDs.

UPSC 2017 Preliminary Exam Q&A Video Discussion with Smart Leaders IAS Page 10 of 39 will be available in our youtube channel “Smart Leaders IAS Online” at 9:00pm

Select the correct answer using the codes  Amarkantak is a Hindu place in given below: Madhya Pradesh, a site where three (a) 1 and 3 only rivers Son, Johila and Narmada start (b) 2 only  Omkareshwar is a (c) 1, 2 and 3 dedicated to God Shiva. It is one of the (d) None of the above statements is correct 12 revered Jyotirlinga shrines of Shiva Explanation: An OLED has a substrate made of clear 26. Consider the following statements: plastic, glass or foil. The substrate supports 1) In the election for Lok Sabha or State the OLED. Assembly, the winning candidate must The plastic, organic layers of an OLED are get at least 50 percent of the votes thinner, lighter and more flexible than the polled, to be declared elected. crystalline layers in an LED or LCD. 2) According to the provisions laid down in Transparent OLEDs have only transparent the Constitution of India, in Lok Sabha, components (substrate, cathode and anode) the Speaker's post goes to the majority and, when turned off, are up to 85 percent party and the Deputy Speaker's to the as transparent as their substrate. When a Opposition. transparent OLED display is turned on, it Which of the statements given above is/are allows light to pass in both directions. correct? Source: (a) 1 only http://electronics.howstuffworks.com/oled (b) 2 only 1.htm (c) Both 1 and 2 http://electronics.howstuffworks.com/oled (d) Neither 1 nor 2 4.htm Explanation: http://electronics.howstuffworks.com/oled A first-past-the-post (abbreviated 5.htm as FPTP, 1stP, 1PTP or FPP) voting method is one in which voters indicate on a 25. Which of the following is/are famous for ballot the candidate of their choice, and the Sun temples? candidate who receives most votes wins. 1) Arasavalli First-past-the-post voting is one of 2) Amarakantak several plurality voting methods. 3) Omkareshwar Under a first-past-the-post voting method Select the correct answer using the code the highest polling candidate (or a group of given below: candidates for some cases) is elected. In this (a) 1 only real-life example, Mr.X obtained a greater (b) 2 and 3 only number than the other candidates, and so (c) 1 and 3 only was declared the winner, even though a (d) 1, 2 and 3 majority of voters did not vote for him. Explanation: The Speaker of the Lok Sabha is the  Arasavalli Sun Temple which is presiding officer of the Lok dedicated to the Hindu solar deity Sabha,[1]the lower house of the Parliament Surya, located in this village of India. The speaker is elected in the very first meeting of the Lok Sabha following general elections. Serving for a term of five

UPSC 2017 Preliminary Exam Q&A Video Discussion with Smart Leaders IAS Page 11 of 39 will be available in our youtube channel “Smart Leaders IAS Online” at 9:00pm

years, the Speaker chosen from amongst the billion in 2003-04. Since 1991, share of members of the Lok Sabha, and is by India’s export in world trade started convention a member of the ruling party or increasing. alliance. Source: The Deputy Speaker of the Lok Sabha is https://www.rbi.org.in/scripts/Publication the vice-presiding officer of the Lok Sabha, ReportDetails.aspx?ID=444 http://statisticstimes.com/ the lower house of the Parliament of India. economy/sectorwise-gdp- contribution-of- S/He acts as the presiding officer in case of india.phphttps://tradingeconomics.com/in leave or absence caused by death or illness dia/exports of the Speaker of the Lok Sabha. It is by convention that position of Deputy Speaker 28. What is the application of Somatic Cell is offered to opposition party in India. Nuclear Transfer Technology? Source: (a) Production of biolarvicides https://en.wikipedia.org/wiki/First-past- (b) Manufacture of biodegradable plastics the-post_voting#Example (c) Reproductive cloning of animals https://en.wikipedia.org/wiki/Speaker_of_t (d) Production of organisms free of diseases he_Lok_Sabha Explanation: https://en.wikipedia.org/wiki/Deputy_Spe Somatic cell nuclear transfer (SCNT), aker_of_the_Lok_Sabha technique in which the nucleus of a somatic (body) cell is transferred to the cytoplasm 27. Which of the following has/have occurred of an enucleated egg (an egg that has had its in India after its liberalization of economic own nucleus removed). Once inside the egg, policies in 1991? the somatic nucleus is reprogrammed by 1) Share of agriculture in GDP increased egg cytoplasmic factors to become a zygote enormously. (fertilized egg) nucleus. 2) Share of India's exports in world trade It can be used in embryonic stem cell increased. research, or in regenerative medicine 3) FDI inflows increased. where it is sometimes referred to as 4) India's foreign exchange reserves "therapeutic cloning." increased enormously. Source: Select the correct answer using the codes https://www.britannica.com/science/soma given below: tic-cell-nuclear-transfer (a) 1 and 4 only (b) 2, 3 and 4 only 29. Consider the following statements:

(c) 2 and 3 only 1) National Payments Corporation of India (NPCI) helps in promoting the financial (d) 1, 2, 3 and 4 inclusion in the country. Explanation: Share of agriculture in GDP is showing a 2) NPCI has launched RuPay, a card decreasing tend. An analysis of the sources payment scheme. of reserves accretion during the entire Which of the statements given above is/are reform period from 1991 onwards reveals correct? that the increase in forex reserves has been (a) 1 only facilitated by an increase in the annual (b) 2 only quantum of foreign direct investment (FDI) (c) Both 1 and 2 from US $ 129 million in 1991-92 to US$ 4.7 (d) Neither 1 nor 2

UPSC 2017 Preliminary Exam Q&A Video Discussion with Smart Leaders IAS Page 12 of 39 will be available in our youtube channel “Smart Leaders IAS Online” at 9:00pm

Explanation: it to increase its foreign exchange National Payments Corporation of India reserves. (NPCI) initiated the launch of RuPay card in 3) It will enormously increase the growth India. and size of economy of India and will Source: enable it to overtake China in the near http://www.npci.org.in/aboutus.aspx# future. Select the correct answer using the codes http://www.business- given below: tandard.com/article/pf/five-things-you- (a) 1 only wanted-to- know-about-the-rupay-card- (b) 2 and 3 only 114080100278_1.html (c) 1 and 3 only (d) 1, 2 and 3 30. The term `M-STrIPES’is sometimes seen in Explanation: the news in the context of Statement 1 is correct. (a) Captive breeding of Wild Fauna GST will reduce the price of the (b) Maintenance of Tiger Reserves manufactured goods thus makes exports (c) Indigenous Satellite Navigation System more competitive. When export increases, (d) Security of National Highways we might have a favourable forex and Explanation: current account deficit. But Current The National Tiger Conservation Authority Account also includes role of imports which (NTCA), which has been upgrading the GST does not cover. As the statements is monitoring mechanism for the large cats, ambiguous, a conclusion cannot be arrived brought in field officials and technical staff certainly. According to various estimates implementing GST might boost the GDP of 11 tiger reserves (TRs) of the country to growth by 80 to 100 basis point. But will it Baripada to introduce them to Monitoring enormously increase the growth and size of System for Tiger-Intensive Protection and the economy? Cannot be said with Ecological Status, known as M-STrIPES. certainty. In near future, it may or may not The android-based monitoring software overtake China’s growth. But that also M-STrIPES will be used across all the TRs depends on China’s performance. As the of the country. statements is ambiguous, a conclusion Source: cannot be arrived certainly. http://www.newindianexpress.com/states /odisha/2017/feb/08/m-stripes-to- 32. 'Broad-based Trade and Investment monitor-str-from-april-1568257.html Agreement (BTIA)' is sometimes seen- in the news in the context of negotiations held 31. What is/are the most likely advantages of between India and implementing 'Goods and Services Tax (a) European Union (GST)'? (b) Gulf Cooperation Council 1) It will replace multiple taxes collected (c) Organization for Economic Cooperation by multiple authorities and will thus and Development create a single market in India. (d) Shanghai Cooperation Organization 2) It will drastically reduce the ‘Current Account Deficit’ of India and will enable

UPSC 2017 Preliminary Exam Q&A Video Discussion with Smart Leaders IAS Page 13 of 39 will be available in our youtube channel “Smart Leaders IAS Online” at 9:00pm

Explanation: accepted the Agreement. India is the 76th Obtaining greater access to the market for WTO member to accept the TFA. services in the European Union (EU) is key WTO members concluded negotiations at for the progress of the Broadbased Trade the 2013 Bali Ministerial Conference on the and Investment Agreement (BTIA) between landmark Trade Facilitation Agreement the EU and India (TFA), which entered into force on 22 But from the period when the negotiations February 2017 following its ratification by for BTIA started in June 2007 till today, the two-thirds of the WTO membership. economic scenario has worsened and Source: become gloomier for both India and the https://www.wto.org/english/news_e/new European Union. s16_e/fac_21apr16_e.htm Source: https://www.wto.org/english/tratop_e/tra http://www.thehindu.com/business/Econo dfa_e/tradfa_e.htm my/market-access-for-indias-services-key- to-btia-nirmala- 34. What is the importance of developing sitharman/article8414503.ece Chabahar Port by India? http://pib.nic.in/newsite/mbErel.aspx?reli (a) India’s trade with African countries will d=87453 enormously increase. (b) India’s relations with oil-producing Arab 33. Consider the following statements : countries will be strengthened. 1) India has ratified the Trade Facilitation (c) India will not depend on Pakistan for Agreement (TFA) of WTO. access to Afghanistan and Central 2) TFA is a part of WTO's Bali Ministerial Asia. Package of 2013. (d) Pakistan will facilitate and protect the 3) TFA came into force in January 2016. installation of a gas pipeline between Select the correct answer using the codes Iraq and India. given below: Explanation: (a) 1 and 2 only No other international port has seen the (b) 1 and 3 only level of involvement and enthusiasm from (c) 2 and 3 only Chabahar as India. The port will make way (d) 1, 2 and 3 for India to bypass Pakistan in transporting Explanation: goods to Afghanistan using a sea-land India has ratified the new Trade Facilitation route. Agreement (TFA). India’s WTO ambassador This will also give momentum to the Anjali Prasad handed over her country’s International North-South Transport instrument of acceptance to Director- Corridor of which both are initial General Roberto Azevêdo on 22 April. signatories along with Russia. Iran is the Concluded at the WTO’s 2013 Bali key gateway in this project. It entails the Ministerial Conference, the TFA contains ship, rail, and road routes for moving provisions for expediting the movement, freight between India, Russia, Iran, Europe release and clearance of goods, including and Central Asia. The route primarily goods in transit. involves moving freight from India, Iran, The TFA will enter into force once two- Azerbaijan and Russia. The objective of the thirds of the WTO membership has formally corridor is to increase trade connectivity

UPSC 2017 Preliminary Exam Q&A Video Discussion with Smart Leaders IAS Page 14 of 39 will be available in our youtube channel “Smart Leaders IAS Online” at 9:00pm

between major cities such as Mumbai, affected by cyber security incident may Moscow, Tehran, Baku, Astrakhan etc. report the incident to CERT-IN”. It would counter Chinese presence in the Source: Arabian sea through the support to Page no. 4 Pakistan in developing Gwadar port. It can http://meity.gov.in/writereaddata/files/G_ be used to station security vessels for merchant ships off the African coast apart S_R%2020%20%28E%292_0.pdf from giving the country a foothold in the western Arabian Sea, which is important as 36. Right to vote and to be elected in India is a many of its energy imports pass through (a) Fundamental Right the route. (b) Natural Right Source: (c) Constitutional Right http://economictimes.indiatimes.com/new (d) Legal Right s/politics-and-nation/five-things-about- Explanation: chabahar-port-and-how-india-gains-from- Article 326. Elections to the House of the it/articleshow/52400399.cms People and to the Legislative Assemblies of

States to be on the basis of adult suffrage. - 35. In India, it is legally mandatory for which of The elections to the House of the People the following to report on cyber security and to the Legislative Assembly of every incidents? State shall be on the basic of adult suffrage 1) Service providers ;that is to say every person who is a citizen 2) Data centres of India and who is not less than eighteen 3) Body corporate years of age on such date as may be fixed in Select the correct answer using the code that behalf by of under any law made by the given below : appropriate Legislature and is not (a) 1 only otherwise disqualified under this (b) 1 and 2 only Constitution or any law made by the (c) 3 only appropriate Legislature on the ground of (d) 1, 2 and 3 non-residence, unsoundness of mind, crime Explanation: or corrupt or illegal practice shall be CERT-IN has today released some entitled to be registered as a voter at any advertisements in news papers reiterating such election. the rules that require mandatory reporting Section 62 of Representation of People Act, of cyber incidents. The circular makes a 1951 :- Right to vote- (1) No person who is reference to the notification dated 16th not, and except as expressly provided by January, 2014 titled “Information this Act, every person who is, for the time Technology (The Indian Computer being entered in the electoral roll of any Emergency Response Team and Manner of constancy shall be entitled to vote in that Performing Functions and Duties) Rules constituency. ,2013” (Copy available here) in which under Source: http://eci.nic.in/archive/handbook/CANDI Section 12(1)(a), it is stated that “Any DATES/cap3/cap3_1.htm individual, organization or corporate entity

UPSC 2017 Preliminary Exam Q&A Video Discussion with Smart Leaders IAS Page 15 of 39 will be available in our youtube channel “Smart Leaders IAS Online” at 9:00pm

37. What is the purpose of 'evolved Laser of the Sarva Abhiyan. Through this Interferometer Space Antenna (eLISA)' initiative people from the Indian Diaspora, project? retired teachers, retired government (a) To detect neutrinos officials including retired defence (b) To detect gravitational waves personnel, retired professionals and (c) To detect the effectiveness .of missile women who are home makers can defence system volunteer at a school that requests for one. (d) To study the effect of solar flares on our Vidyanjali will also cover initiatives under communication systems the Corporate Social Responsibility (CSR) Explanation: and Public Private Partnership (PPP), with The Laser Interferometer Space Antenna Public Sector Undertaking (PSU) (LISA) is a joint NASA-ESA project to Companies, private corporate and others. develop and operate a space-based A very large number of people who are not gravitational wave detector. LISA detects part of the government have expressed a gravitational-wave induced strains in desire to give back to society and space-time by measuring changes of the particularly to children in government separation between fiducial masses in three schools. This includes retired government spacecraft 5 million kilometers apart. and other professionals, home makers and Source: also members of the Indian Diaspora and https://lisa.nasa.gov/ their children.

At the same time, there is a huge gap in the 38. What is the purpose of Tidyanjali Yojana'? provisioning of educational services to 1) To enable the famous foreign children in government schools particularly educational institutions to open their for co-scholastic areas. campuses in India. This programme has been envisaged to 2) To increase the quality of education bring together people willing to volunteer provided in government schools by their services at schools which really need taking help from the private sector and them. the community. The aim of the programme is to strengthen 3) To encourage voluntary monetary implementation of co-scholastic activities in contributions from private individuals government schools through services of and organizations so as to improve the infrastructure facilities for primary and volunteers. secondary schools. the volunteers will not teach any subject Select the correct answer using the codes in school or take regular class. given below: The purpose of a Volunteer at a school is to (a) 2 only strengthen and enrich the implementation (b) 3 only of co-scholastic activities. Volunteers (c) 1 and 2 only would be able to bring diverse talents and (d) 2 and 3 only skills, thereby making valuable contribution Explanation: to the learning process at school. Vidyanjali is an initiative to enhance Volunteer sunder community and private sector involvement he Vidyanjali programme will offer their in Government run elementary schools services to government schools without across the country under the overall aegis receiving compensation. The Volunteers UPSC 2017 Preliminary Exam Q&A Video Discussion with Smart Leaders IAS Page 16 of 39 will be available in our youtube channel “Smart Leaders IAS Online” at 9:00pm

will conduct one or more from among the Explanation: following co-scholastic activities with Unnat Bharat Abhiyan is inspired by the children in standards 1to 8. vision of transformational change in rural  reading to children; development processes by leveraging  helping children with creative writing; knowledge institutions to help build the  public speaking; architecture of an Inclusive India. Their  play acting; mission is conceptualised as a movement to  preparing story books with children; enable processes that connect institutes of  There will be no fees charged from the higher education with local communities to Volunteer at any stage of the address the development challenges of programme. rural India through participatory processes  In case of all girls’ schools, only female and appropriate technologies for volunteers would be allowed. accelerating sustainable growth. It also  The volunteers are requested not to aims to create a virtuous cycle between the offer gifts to individual students or society and an inclusive university system teachers. Only very small and by providing knowledge and practices for inexpensive gifts which are in the nature emerging professions and to upgrade the of mementos would be allowed to be capabilities of both the public and the received from the volunteer, but these private sectors. Source: need to be handed over to the Head http://unnat.iitd.ac.in/index.php/en/frame Teacher. work/introduction Source:

http://vidyanjali.mygov.in/index.php/front 40. Consider the following statements : end/guideline 1) The Election Commission of India is a

five-member body. 39. What is the aim of the programme `Unnat 2) Union Ministry of Home Affairs decides Bharat Abhiyan'? the election schedule for the conduct of (a) Achieving 100% literacy by promoting both general elections and bye- collaboration between voluntary elections. organizations an government's education 3) Election Commission resolves the system and local communities. disputes relating to splits/mergers of (b) Connecting institutions of higher recognised political parties. education with local communities to Which of the statements given above is/are address development challenges correct? through appropriate technologies. (a) 1 and 2 only (c) Strengthening India's scientific research (b) 2 only institutions in order to make India a (c) 2 and 3 only scientific and technological power. (d) 3 only (d) Developing human capital by allocating Explanation: special funds for health care and At present, the Election Commission of education of rural and urban poor, and India is a three-member body, with one organizing skill development Chief Election Commissioner and two programmes and vocational training for them. Election Commissioners.

UPSC 2017 Preliminary Exam Q&A Video Discussion with Smart Leaders IAS Page 17 of 39 will be available in our youtube channel “Smart Leaders IAS Online” at 9:00pm

It was not a multi member body from the (c) It is endemic to a particular region of beginning. It was a single - member body India. when it was first set up in 1950 and up to (d) Both (b) and (c) stated above are correct 15th October, 1989 with only the Chief in this context. Election Commissioner. From 16th October, Explanation: 1989 upto the 1st January, 1990, it became The Schedule 1 of the Wildlife Protection a three-member body with R.V.S.Peri Sastri Act 1972 ensures the maximum legal (C.E.C) and S.S.Dhanoa and V.S.Seigell as protection for wild animals. As Tiger is a Election Commissioners. From 2nd January, schedule 1 animal under the WPA, 1972, a 1990 to 30th September, 1993, it was a species of tortoise will enjoy the same single-member Commission and again from 1st October, 1993 it has become a three- status as Tiger. member Commission. Source: The Election Commission decides the Page no.9 and 48 schedule for election. http://nbaindia.org/uploaded/Biodiversity The Commission normally announces the india/Legal/15.%20Wildlife%20%28Prote schedule of elections in a major Press ction%29%20Act,%201972.pdf Conference a few weeks before the formal process is set in motion. 42. In India, Judicial Review implies Splits, mergers and alliances have (a) the power of the Judiciary to frequently disrupted the compositions of pronounce upon the constitutionality political parties. This has led to a number of of laws and executive orders. disputes over which section of a divided (b) the power of the Judiciary to question the party gets to keep the party symbol, and wisdom of the laws enacted by the how to classify the resulting parties in Legislatures. terms of national and state parties. The (c) the power of the Judiciary to review all Election Commission has to resolve these the legislative enactments before they are disputes, although its decisions can be assented to by the President. challenged in the courts. (d) the power of the Judiciary to review its Source: own judgements given earlier in similar http://eci.nic.in/eci_main1/the_function.as or different cases. px#splitsandmergers Explanation: Judicial review is the power of a court to 41. In India, if a species of tortoise is declared enquire whether a law, executive order or protected under Schedule I of the Wildlife other official action conflicts with written (Protection) Act, 1972, what does it imply ? constitution and , if the court concludes that (a) It enjoys the same level of protection it does, declare it unconstitutional and as the tiger. void”. (b) It no longer exists in the wild, a few Source: individuals are under captive protection; http://www.legalservicesindia.com/article /article/judicial-review-in-india-and-usa- and now it is impossible to prevent its 1734-1.html extinction.

UPSC 2017 Preliminary Exam Q&A Video Discussion with Smart Leaders IAS Page 18 of 39 will be available in our youtube channel “Smart Leaders IAS Online” at 9:00pm

43. With reference to Indian freedom struggle, (b) Kuno Palpur Wildlife Sanctuary consider the following events : (c) Mudumalai Wildlife Sanctuary 1) Mutiny in Royal Indian Navy (d) Sariska National Park 2) Quit India Movement launched Explanation: 3) Second Round Table Conference The Asiatic Lion Reintroduction Project is What is the correct chronological sequence an initiative of the Indian Government to of the above events? provide safeguards to the Asiatic lion (a) 1-2-3 (Panthera leo persica) from extinction in (b) 2-1-3 the wild by means of reintroduction. The (c) 3-2-1 last wild population of the Asiatic lion is (d) 3-1-2 found in the Gir Forest region of the state of Explanation: Gujarat. The single population faces the  RIN - 18–23 February 1946 threats of epidemics, natural disasters and  Quit India Movement - 8 August 1942 other anthropogenic factors.  Second Round Table Conference - The process of translocation of a few Asiatic December 1931 lions from Gir National Park to Palpur Kuno is a “long-term action. 44. Consider the following statements: The whole programme encompasses action 1) Tax revenue as a percent of GDP of India for over 25 years. The Ministry of has steadily increased in the last decade. Environment and Forest has constituted an 2) Fiscal deficit as a percent of GDP of India Experts Committee for planning and has steadily increased in the last decade. implementation of translocation of Asiatic Which of the statements given above is/are lions from Gir to Kuno. The group includes representatives of Madhya Pradesh and correct? Gujarat (a) 1 only Source: (b) 2 only http://www.livemint.com/Politics/niSTqjT (c) Both 1 and 2 WnYXmlhogS55tuM/Gir-Lions-Forest- (d) Neither 1 nor 2 ministry-cites-challenges-in-Madhya- Explanation: Prades.html Fiscal Deficit as percent of GDP is getting decreased. 46. Which of the following are not necessarily Source: the consequences of the proclamation of the http://economictimes.indiatimes.com/new President's rule in a State ? s/economy/policy/eco-survey-2016-raise- 1) Dissolution of the State Legislative taxpayers-to-gdp-ratio-not-exemption- Assembly threshold/articleshow/51153610.cms 2) Removal of the Council of Ministers in http://pibphoto.nic.in/documents/graphic the State

/gbig298.JPG 3) Dissolution of the local bodies Select the correct answer using the code

given below 45. Recently there was a proposal to (a) 1 and 2 only translocate some of the lions from their (b) 1 and 3 only natural habitat in Gujarat to which one of (c) 2 and 3 only the following sites? (d) 1, 2 and 3

(a) Corbett National Park UPSC 2017 Preliminary Exam Q&A Video Discussion with Smart Leaders IAS Page 19 of 39 will be available in our youtube channel “Smart Leaders IAS Online” at 9:00pm

Explanation: labour are prohibited and any The proclamation of president’s rule under contravention of this provision shall be an Article 356 necessarily results in the offence punishable in accordance with law removal of council of ministers in the state, Articles 24 of Indian constitution says No but not necessarily the dissolution of state child below the age of fourteen years shall assembly or local bodies. be employed to work in any factory or mine As per the provisions of Article 356 of or engaged in any other hazardous Indian Constitution, the state Assembly can employment. be dissolved when any of the following factors prevent the state government from 48. Which of the following is geographically functioning as per the Constitution: closest to Great Nicobar? When the state Assembly fails to form a (a) Sumatra government and elect a leader as Chief (b) Borneo Minister Whenever there’s a breakdown of (c) Java a coalition If Assembly elections are (d) Sri Lanka postponed for unavoidable reasons Explanation: Insurgencies and internal subversions Prevention or facilitation of bifurcation of states.

Source:

http://www.elections.in/political-

corner/when-and-why-is-a-state-assembly- dissolved/

47. Which of the following are envisaged by the Right against Exploitation in the Constitution of India? 1) Prohibition of traffic in human beings and forced labour 2) Abolition of untouchability 49. Out of the following statements, choose the 3) Protection of the interests of minorities one that brings out the principle underlying 4) Prohibition of employment of children the Cabinet form of Government : in factories and mines (a) An arrangement for minimizing the Select the correct answer using the code criticism against the Government whose given below: responsibilities are complex and hard to (a) 1, 2 and ,4 only carry out to the satisfaction of all (b) 2, 3 and 4 only (b) A mechanism for speeding up the (c) 1 and 4 only (d) 1, 2, 3,ancl 4 activities of the Government whose Explanation: responsibilities are increasing day by day. Right against exploitation is provided (c) A mechanism of parliamentary - under Articles 23 and 24 of Indian democracy for ensuring collective constitution. Article 23. provides for responsibility of the Government to Prohibition of traffic in human beings and the people. forced labour (d) A device for strengthening the hands of Article 23(1) says Traffic in human beings the head of the Government whose hold and begar and other similar forms of forced over the people is in a state of decline. UPSC 2017 Preliminary Exam Q&A Video Discussion with Smart Leaders IAS Page 20 of 39 will be available in our youtube channel “Smart Leaders IAS Online” at 9:00pm

Explanation: (c) The federating units have been given Collective Ministerial Responsibility in the unequal representation in the Rajya sole crux of Parliamentary democracy. The Sabha. principle of collective responsibility (d) It is the result of an agreement among represents ministerial accountability to the the federating units. legislature. In India, the doctrine of Explanation: collective responsibility of the Union Indian constitution is said to be federal, Executive to the House of the People and of then why not India is a federation but Union. It is mainly for two reasons viz. (a) the State Executive to the Legislative that the Indian Union is not the result of an Assembly is specifically enshrined in the agreement by the states and (b) the Constitution. Article 75(3) lays down that component states have no freedom to the Council of Ministers shall be collectively secede from it. responsible to the Lok Sabha. It means that Source: the Government must maintain a majority http://www.legallyindia.com/views/entry/ in the Lok Sabha as a condition of its india-union-or-federation survival. The object of Collective responsibility is to make the whole body of 51. The object of the Butler Committee of 1927 persons holding ministerial office was to collectively, or, if one may so put it, (a) Define the jurisdiction of the Central and “vicariously responsible for such acts of the Provincial Governments. others as are referable to their collective (b) Define the powers of the Secretary of violation so that, even if an individual may State for India. not be personally responsible for it, yet, he (c) Impose censorship on national press. will be deemed to share the responsibility (d) Improve the relationship between the with those who may have actually Government of India and the Indian States. committed some wrong.” Explanation: Lord Salisbury explained the principle of  Indian states committee appointed a collective responsibility as: “For all that committee under the Chairmanship of Sir passes in the Cabinet, each member of it who Harcourt Butler was to Investigate, does not resign is absolutely irretrievably Improve and clarify the relationship responsible, and has no right afterwards to between the paramount power and the say that he agreed in one sense to a Princes of Princely States compromise while in another he was

persuaded by his colleagues. ” 52. The term 'Domestic Content Requirement' Source: is sometimes seen in the news with https://rostrumlegal.com/the-concept-of- reference to collective-ministerial-responsibility-in- (a) Developing solar power production in india-theory-practice/ our country (b) Granting licenses to foreign T.V. channels 50. Which one of the following is not a feature in our country of Indian federalism? (c) Exporting our food products to other (a) There is, an independent judiciary in countries India. (d) Permitting foreign educational (b) Powers have been clearly divided institutions to set up their campuses in between the Centre and the States. our country UPSC 2017 Preliminary Exam Q&A Video Discussion with Smart Leaders IAS Page 21 of 39 will be available in our youtube channel “Smart Leaders IAS Online” at 9:00pm

Explanation: The United States will host the fourth The domestic content requirement (DCR) Nuclear Security Summit in Washington, category, the mandate for solar projects in D.C., on March 31 and April 1, 2016. India to utilize domestically manufactured The International Panel on Fissile Materials solar modules and cells, was instituted in (IPFM) was founded in January 2006 and is the Jawaharlal Nehru National Solar an independent group of arms-control and Mission (JNNSM) from the beginning of nonproliferation experts from both nuclear 2010 in an effort to create a healthy and weapon and non-nuclear weapon states. robust indigenous manufacturing base and The Panel has been co-chaired since 2015 to elevate India’s status as a solar hub. Source: by Professor Alexander Glaser and Dr. Zia http://economictimes.indiatimes.com/artic Mian of Princeton University and Professor leshow/58398282.cms?utm_source=conten Tatsujiro Suzuki of Nagasaki University, tofinterest&utm_medium=text&utm_campa Japan. Previously, it was co-chaired by ign=cppst Professor Jose Goldemberg of the University of Sao Paolo, Brazil (2006-2007), 53. Consider the following statements: Dr. R. Rajaraman (2007-2014) Professor 1) The Nuclear Security Summits are Emeritus, of Jawaharlal Nehru University, periodically held under the aegis of the New Delhi, India, and Professor Frank von United Nations. Hippel of Princeton University (2006- 2) The International Panel on Fissile 2014). Materials is an organ of International Its members include nuclear experts from Atomic Energy Agency. seventeen countries: Brazil, Canada, China, Which of the statements given above is/are France, Germany, India, Iran, Japan, Mexico, correct? Norway, Pakistan, South Korea, Russia, (a) 1 only South Africa, Sweden, the United Kingdom, (b) 2 only and the United States. This group of (c) Both 1 and 2 countries includes seven nuclear-weapon (d) Neither 1 nor 2 states and ten non-weapon states Explanation: Source: In his 2009 Prague speech, President http://www.nss2016.org/ Obama stated that nuclear terrorism “is the http://fissilematerials.org/ipfm/about.html most immediate and extreme threat to 54. Who among the following can join the global security.” To mitigate this threat, the President urged that “we act with purpose National Pension System (NPS) ?

and without delay,” announcing “a new (a) Resident Indian citizens only

international effort to secure vulnerable (b) Persons of age from 21 to 55 only

nuclear material around the world” that (c) All State Government employees joining would begin with “a Global Summit on the services after the date of notification Nuclear Security that the United States will by the respective State Governments

host. The first Nuclear Security Summit was (d) All Central Government employees held in Washington, DC in 2010, and was including those of Armed Forces joining t followed by additional Summits in Seoul in the services on or after 1s April, 2004 2012 and The Hague in 2014.

UPSC 2017 Preliminary Exam Q&A Video Discussion with Smart Leaders IAS Page 22 of 39 will be available in our youtube channel “Smart Leaders IAS Online” at 9:00pm

55. With reference to river Teesta, consider the (a) 1 only following statements : (b) 2 only 1) The source of river Teesta is the same as (c) Both 1 and 2 that of l3rahmaputra but it flows (d) Neither 1 nor 2 through Sikkim. Explanation: 2) River Rangeet originates in Sikkim and Zika virus disease is caused by a virus it is a tributary of river Teesta.. transmitted primarily by Aedes mosquitoes. 3) River Teesta flows into Bay of Bengal on Aedes aegypti, the yellow fever mosquito, is the border of India and Bangladesh. a mosquito that can spread dengue fever, Which of the statements given above is/are chikungunya, Zika fever, Mayaro and yellow correct? fever viruses, and other diseases. (a) 1 and 3 only Zika virus is primarily transmitted to (b) 2 only people through the bite of an infected (c) 2 and 3 only mosquito from the Aedes genus, mainly (d) 1, 2 and 3 Aedes aegypti in tropical regions. Sexual transmission of Zika virus is also possible. Other modes of transmission such as blood Explanation: transfusion are being investigated. Statement 1 is wrong Source: http://www.who.int/mediacentre/factshee  The source of Teesta River is: located in ts/zika/en/ the state of Sikkim

 Brahmaputra River originates on the 57. Consider the following statements: Angsi Glacier located on the northern 1) The Standard Mark of Bureau of Indian side of the Himalayas in Burang County Standards (BIS) is mandatory for of Tibet automotive tyres and tubes. Statement 2 is correct 2) AGMARK is a quality Certification Mark  The Rangeet River is a tributary of the issued by the Food and Agriculture Teesta river, which is the largest river in Organization (FAO). the Indian state of Sikkim. Which of the given above statement(s)  It originates in the Himalayan mountains in West Sikkim district is/are correct? Statement 3 is wrong (a) 1 only  River Teesta meets the Bay of Bengal (b) 2 only from Bangladesh after joining (c) Both 1 and 2 Brahmaputra (Jamuna) (d) Neither 1 nor 2 Explanation: 56. Consider the following statements: A certification from the Bureau of 1) In tropical regions, Zika virus disease is Indian Standards (BIS) is mandatory transmitted by the same mosquito that for all types of automotive tyres and transmits dengue. tubes from May 18,2010. 2) Sexual transmission of Zika virus As per the directive of the BIS, all types disease is possible. of tyres and tubes for two-wheelers, Which of the statements given above is/are three-wheelers, passenger cars and correct?

UPSC 2017 Preliminary Exam Q&A Video Discussion with Smart Leaders IAS Page 23 of 39 will be available in our youtube channel “Smart Leaders IAS Online” at 9:00pm

commercial vehicles would now need mandatory-for-tyres-and- BIS marking to be sold in the city. tubes/article12079528.ece AGMARK is a certification mark employed http://agritech.tnau.ac.in/amis/food_agma on agricultural products in India, assuring rk.html that they conform to a set of standards approved by the Directorate of Marketing 58. What is/are the advantage/advantages of and Inspection, an agency of implementing the 'National Agriculture the Government of India. The AGMARK is Market' scheme? legally enforced in India by the Agricultural 1) It is a pan-India electronic trading portal Produce (Grading and Marking) Act of 1937 for agricultural commodities. (and ammended in 1986). The 2) It provides the farmers access to present AGMARK standards cover quality nationwide market, with prices guidelines for 205 different commodities commensurate with the quality of their spanning a variety produce. of Pulses, Cereals, Essential Oils, Vegetable Which of the given above statement(s) Oils, Fruits & Vegetables, and semi- is/are correct? processed products like Vermicelli. (a) 1 only The term agmark was coined by joining the (b) 2 only words 'Ag' to mean agriculture and 'mark' (c) Both 1 and 2 for a certification mark. This term was (d) Neither 1 nor 2 introduced originally in the bill presented Explanation: in the parliament of India for National Agriculture Market (NAM) is a the Agricultural Produce (Grading and pan-India electronic trading portal which Marking) Act. networks the existing APMC mandis to Agmark Laboratories create a unified national market for The Agmark certification is employed agricultural commodities. through fully state-owned Agmark NAM addresses these challenges by creating laboratories located across the nation a unified market through online trading which act as testing and certifying centres. platform, both, at State and National level In addition to the Central AGMARK and promotes uniformity, streamlining of Laboratory (CAL) in Nagpur, there procedures across the integrated markets, are Regional AGMARK Laboratories removes information asymmetry between (RALs) in 11 nodal cities (Mumbai, New buyers and sellers and promotes real time Delhi, Chennai, Kolkata, Kanpur, Kochi, Gunt price discovery, based on actual demand ur, Amritsar, Jaipur, Rajkot, Bhopal). Each of and supply, promotes transparency in the regional laboratories is equipped with auction process, and access to a and specializes in the testing of products of nationwide market for the farmer, with regional significance. Hence the product prices commensurate with quality of his range that could be tested varies across the produce and online payment and centres. availability of better quality produce and at Source: more reasonable prices to the consumer. http://www.thehindu.com/news/cities/ch Source: ennai/BIS-certification-to-be-made- http://www.enam.gov.in/NAM/home/abou t_nam.html#

UPSC 2017 Preliminary Exam Q&A Video Discussion with Smart Leaders IAS Page 24 of 39 will be available in our youtube channel “Smart Leaders IAS Online” at 9:00pm

59. With reference to the 'National Intellectual (d) 1, 2 and 3 Property Rights Policy', consider the Explanation: following statements : The Government of India accorded the 1) It reiterates India's commitment to the highest level of protection to Gharial by Doha Development Agenda and the bringing it under Schedule I of the Wild Life TRIPS Agreement. Protection Act, 1972. In 1976, Project 2) Department of Industrial Policy and Crocodile was initiated with support from Promotion is the nodal agency for the United Nations Development Programme and Food and Agriculture regulating intellectual property rights in Organization. India. The Indian wild ass is a species with a small Which of the given above statement(s) distribution range restricted to the salt is/are correct? desert in the Little Rann of Kutch. Due to (a) 1 only the various threats faced by Indian wild ass (b) 2 only has been listed as EN Endangered by the (c) Both 1 and 2 IUCN Red List and in Schedule I of the (d) Neither 1 nor 2 Wildlife Protection Act (1972) Explanation: The Indian wild buffalo (Bubalus bubalis L.) 'National Intellectual Property Rights has been listed as an endangered species in Policy', reiterates India’s commitment to the Red Data book of IUCN and categorized the Doha Development Agenda and the in appendix III of CITES. It is classified in TRIPS agreement. 'National Intellectual schedule I of Indian wildlife (Protection) act Property Rights Policy', suggests making 1972, which indicates that it has been given the department of industrial policy and top priority for conservation. Wild buffalo declared as State Animal of Chhattisgarh promotion (DIPP) the nodal agency for all state in the year of 2001. IPR issues. Source: Source: http://www.moef.nic.in/downloads/public http://pib.nic.in/newsite/PrintRelease.asp -information/Gharial%20booklet_MoEF.pdf x?relid=145338 http://cza.nic.in/wild%20Ass%20Studboo http://www.thehindu.com/business/all- k.pdf you-need-to-know-about-the-intellectual- http://envfor.nic.in/legis/wildlife/wildlife2 property-rights-policy/article8600530.ece s1.pdf

60. According to the Wildlife (Protection) Act, 61. Which of the following statements is/are 1972, which of the following animals true of the Fundamental Duties of an Indian cannot be hunted by any person except citizen? under some provisions provided by law ? 1) A legislative process has been provided 1) Gharial to enforce these duties 2) Indian wild ass 2) They are correlative to legal duties. 3) Wild buffalo Select the correct answer using the code Select the correct answer using the codes given below: given below: (a) 1 only (a) 1 only (b) 2 only (b) 2 and 3 only (c) Both 1 and 2 (c) 1 and 3 only (d) Neither 1 nor 2

UPSC 2017 Preliminary Exam Q&A Video Discussion with Smart Leaders IAS Page 25 of 39 will be available in our youtube channel “Smart Leaders IAS Online” at 9:00pm

Explanation: Radhakanta Deb There is no legal sanction provided for  Radhakanta Deb was a scholar and a violation or non-performance of leader of the Calcutta conservative Fundamental Duties. There is neither Hindu society specific provision for enforceability nor any  He was founder-president British Indian specific prohibition. Association in 1851, a position he held Article 29 (1) of the Universal till his death Declaration of Human rights, 1948,  He helped David Hare and funded states: founding of the Hindu College in “Everyone has duties to the community in Calcutta which alone the free and full development Gazulu Lakshminarasu Chetty of his personality is possible.”  He was an Indian merchant, Indian It is the basic principle of jurisprudence independence activist and political that every right has a correlative duty and activist who founded the Madras Native every duty has a correlative Right. But the Association and the first Indian-owned rule is not absolute. It is subject to certain newspaper, The Crescent. exceptions in the sense that a person may  On the other hand, Madras Mahajana have a right, but there may not be a Sabha was established in May 1884. by correlative duty. The fundamental Duties M. Veeraraghavachariar, G. Subramania are not enforceable by mandamus or any Iyer and P. Anandacharlu other legal remedy Surendranath Banerjee Source:  He was one of the earliest Indian http://cslra.in/blog/2015/11/13/fundame political leaders during the British Raj. ntal-duties-under-the-constitution-as- He founded the Indian National legally-enforceable-duties-under-different- Association statutes/  He was editor of "The Bengali" newspaper 62. Which of the following statements is/are correct? 63. Which one of the following objectives is not 1) Radhakanta Deb — First President of embodied in the Preamble to the the British Indian Association Constitution of India? 2) Gazulu Lakshminarasu Chetty — (a) Liberty of thought Founder of the Madras Mahajana Sabha (b) Economic liberty 3) Surendranath Banerjee — Founder of (c) Liberty of expression the Indian Association (d) Liberty of belief Which of the above pairs is/are correctly Explanation: matched? Preamble provides for LIBERTY of thought, (a) 1 only expression, belief, faith and worship; (b) 1 and 3 only Source: (c) 2 and 3 only http://www.constitution.org/cons/india/p (d) 1, 2 and 3 reamble.html

UPSC 2017 Preliminary Exam Q&A Video Discussion with Smart Leaders IAS Page 26 of 39 will be available in our youtube channel “Smart Leaders IAS Online” at 9:00pm

64. With reference to 'Quality Council of India The objectives of setting up of small finance (QCI)', consider the following statements : banks will be to further financial inclusion 1) QCI was set up jointly by the by (a) provision of savings vehicles, and (ii) Government of India and the Indian supply of credit to small business units; Industry. small and marginal farmers; micro and 2) Chairman of QCI is appointed by the small industries; and other unorganised Prime Minister on the recommendations sector entities, through high technology- of the industry to the Government. low cost operations. Which of the given above statement(s) Source: is/are correct? https://rbi.org.in/scripts/BS_PressRelease (a) 1 only Display.aspx?prid=32614 (b) 2 only (c) Both 1 and 2 66. With reference to ‘Asia Pacific Ministerial (d) Neither 1 nor 2 Conference on Housing and Urban Explanation: Development (APMCHUD)’, consider the The Quality Council of India (QCI) is a following statements: pioneering experiment of the 1) The first APMCHUD was held in India in Government of India in setting up 2006 on the theme 'Emerging Urban organizations in partnership with the Forms — Policy Responses and Indian industry. Governance Structure'. QCI is governed by a Council of 38 members 2) India hosts all the Annual Ministerial with equal representations of government, Conferences in partnership with ADB, industry and consumers. Chairman of QCI is APEC and ASEAN. appointed by the Prime Minister on Which of the given above statement(s) recommendation of the industry to the is/are correct? government. (a) 1 only Source (b) 2 only http://www.qcin.org/about-qci.php (c) Both 1 and 2 (d) Neither 1 nor 2 65. What is the purpose of setting up of. Small Explanation: Finance Banks (SFBs) in India ? First conference was held in 2006 in New 1) To supply credit to small business units Delh.The conference theme was , “A vision 2) To supply credit to small and marginal for sustainable urbanisation in the Asia- farmers Pacific by 2020”, 3) To encourage young entrepreneurs to The Conference of APMCHUD is a biennial set up business particularly in rural event. This event is hosted by one of the areas. member countries whose offer is consented Select the correct answer using the code to by the members in the previous given below 2nd conference of the APMCHUD was held in (a) 1 and 2 only May, 2008 inTehran, Iran. (b) 2 and 3 only 3rd conference of the APMCHUD was held (c) 1 and 3 only in 2010 in Indonesia (d) 1, 2 and 3 4 th conference of the APMCHUD was held Explanation: in 2012 Amman ,Jordan

UPSC 2017 Preliminary Exam Q&A Video Discussion with Smart Leaders IAS Page 27 of 39 will be available in our youtube channel “Smart Leaders IAS Online” at 9:00pm

5th conference of the APMCHUD was held in your money remains in the bank account 2014 in Seoul. and keeps earning the interest. Digital 6 th conference of the APMCHUD was held wallets don’t give any interest. in 2016 in New Delhi Source: Source: http://www.npci.org.in/UPI_Background.as http://www.apmchud.com/PDF/1st%20co px nference%20proceedings.pdf https://upipayments.co.in/upi-benefits/ http://www.apmchud.com/PDF/guidelines -apmc.pdf 69. The terms 'Event Horizon', 'Singularity', `String Theory' and 'Standard Model' are

67. Democracy's superior virtue lies in the fact sometimes seen in the news in the context that it calls into activity of (a) the intelligence and character of (a) Observation and understanding of the ordinary men and women. Universe (b) the methods for strengthening executive leadership. (b) Study of the solar and the lunar eclipses (c) a superior individual with dynamism and (c) Placing satellites in the orbit of the Earth vision. (d) Origin and evolution of living organisms (d) a band of dedicated party workers. on the Earth Explanation: 68. Which of the following is a most likely In the centre of a black hole is a consequence of implementing the 'Unified gravitational singularity, a one-dimensional Payments Interface (UPI)' ? point which contains a huge mass in an (a) Mobile wallets will not be necessary infinitely small space, where density and for online payments. gravity become infinite and space-time (b) Digital currency will totally replace the curves infinitely, and where the laws of physical currency in about two decades. physics as we know them cease to operate.

(c) FDI inflows will drastically increase. the subatomic composition of the universe

(d) Direct transfer of subsidies to poor people is summarized in what is known as the will become very effective. Standard Model of particle physics. It Explanation: describes both the fundamental building Unified Payments Interface (UPI) is a blocks out of which the world is made, and system that powers multiple bank accounts the forces through which these blocks into a single mobile application (of any interact. participating bank), merging several In the last few decades, string theory has banking features, seamless fund routing & emerged as the most promising candidate merchant payments into one hood. for a microscopic theory of gravity. it Universal Application for transaction. Single attempts to provide a complete, unified, and Application for accessing different bank consistent description of the fundamental accounts structure of our universe. (For this reason it UPI would Replace Many Digital Wallets is sometimes, quite arrogantly, called a The UPI would end this hassle as well. The 'Theory of Everything'). Source: payment through the UPI is as easy as http://www.nuclecu.unam.mx/~alberto/p payment through the wallets. But a single hysics/string.html app is enough to pay anywhere. Also, the

UPI does not ask money in advance. Rather, UPSC 2017 Preliminary Exam Q&A Video Discussion with Smart Leaders IAS Page 28 of 39 will be available in our youtube channel “Smart Leaders IAS Online” at 9:00pm

70. With reference to agriculture in India, how (a) the executive and legislature work can the technique of `genome sequencing', independently. often seen in the news, be ' used in the (b) it provides continuity of policy and is immediate future ? more efficient. 1) Genome sequencing can be used to (c) the executive remains responsible to identify genetic markers for disease the legislature. resistance and drought tolerance in (d) the head of the government cannot be various crop plants. changed without election. 2) This technique helps in reducing the Explanation: time required to develop new varieties of crop plants. The executive branch is dependent upon 3) It can be used to decipher the host- the direct or indirect support of pathogen relationships in crops. the legislative branch and often includes Select the correct answer using the codes members of the legislature. given below: In the Parliamentary system, the executive (a) 1 only is responsible and accountable to the (b) 2 and 3 only legislature for all its actions since it has the (c) 1 and 3 only right to seek detailed information about the (d) 1, 2 and 3 working of the Ministers. The Council of Explanation: Ministers remain in office as long as they In agriculture, genomics aids to: enjoy the support and confidence of the Lok  Improve and design crops with Sabha, i.e., the House of the People. enhanced resistance to factors that Source: influence their growth such as pests, https://www.lawctopus.com/academike/is diseases, drought, frost, floods and so -parliamentary-form-of-government-good- on. enough-for-india-need-for-a-change/  To produce large quantities food on the limited amount of land and to cultivate 72. In the context of India, which one of the more nutrient-rich foods following is the correct relationship  To breed disease resistant, superior between Rights and Duties ? quality livestock and to produce healthy (a) Rights are correlative with Duties. herd. (b) Rights are personal and hence  Genomics helps to accurately predict the independent of society and Duties. genetic merit, to sharpen selective (c) Rights, not Duties, are important for the breeding, enhance desirable traits and advancement of the personality of the increase animal health and welfare. citizen. Source: (d) Duties, not Rights, are important for the http://www.biotecharticles.com/Biotech- stability of the State. Research-Article/Genomics-and-Its- Explanation: Applications-in-Agriculture-2164.html Rights and duties are co relatives. If there https://www.ncbi.nlm.nih.gov/pmc/article are duties towards the public, there are s/PMC4807965/ rights as well. There can be no duty unless there is some person to whom that duty is 71. The main advantage of the parliamentary due. Every right or duty involves a bond of form of government is that obligation. UPSC 2017 Preliminary Exam Q&A Video Discussion with Smart Leaders IAS Page 29 of 39 will be available in our youtube channel “Smart Leaders IAS Online” at 9:00pm

In Minerva mills ltd v. union of India ,The States within India through which you can Supreme Court observed that there may be travel, including the origin and the a rule which imposes an obligation on an destination? individual or authority, and yet it may not (a) 6 be enforceable in court of law, and (b) 7 therefore not give rise to a corresponding (c) 8 enforceable right in another person. But it (d) 9 would still be a legal rule because it Explanation: prescribes a norm of conduct to be followed  Nagaland, Assam, West Bengal, Odisha, by such individual or authority. The law Andhra Pradesh, Tamil Nadu, Kerala may provide a mechanism for enforcement of this obligation, but the existence of the 75. The Parliament of India exercises control obligation does not depend upon the over the functions of the Council of creation of such mechanism. The obligation Ministers through exists prior to and independent of, the 1) Adjournment motion mechanism of enforcement. 2) Question hour Source: 3) Supplementary questions https://www.legalcrystal.com/blog/constit Select the correct answer using the codes utional-law/jurisprudence-relationship- given below: between-rights-and-duties (a) 1 only (b) 2 and 3 only 73. The mind of the makers of the Constitution (c) 1 and 3 only of India is reflected in which of the (d) 1, 2 and 3 following ? Explanation: (a) The Preamble During the law making process, members of (b) The Fundamental Rights the legislature get an opportunity to (c) The Directive Principles of State Policy deliberate on the policy direction of the (d) The Fundamental Duties executive and the ways in which policies Explanation: are implemented. Apart from deliberating The Preamble of the Constitution like the on bills, control may also be exercised preamble of any statute furnish the key to during the general discussions in the House. open the mind of the makers of The Question Hour, which is held every day the Constitution more so because the during the sessions of Parliament, where Constituent Assembly took great pains in Ministers have to respond to searching formulating it so that it may reflect the questions raised by the members; Zero essential features and basic objectives of Hour where members are free to raise any the Constitution. matter that they think is important (though Source: the ministers are not bound to reply), half- http://www.legalservicesindia.com/article an – hour discussion on matters of public /article/role-of-preamble-interpretation- importance, adjournment motion etc. are with-indian-constitution-1390-1.html some instruments of exercising control Source: 74. If you travel by road from Kohima to http://www.ncert.nic.in/ncerts/l/keps205. Kottayam, what is the minimum number of pdf

UPSC 2017 Preliminary Exam Q&A Video Discussion with Smart Leaders IAS Page 30 of 39 will be available in our youtube channel “Smart Leaders IAS Online” at 9:00pm

76. With reference to the Parliament of India, private-members-bills-passed-since- consider the following statements : independence-115042400948_1.html 1) A private member's bill is a bill http://164.100.47.132/LssNew/abstract/p presented by a Member of Parliament rivate_members.htm who is not elected but only nominated 77. With reference to the difference between by the President of India. the culture of Rigvedic Aryans and Indus 2) Recently, a private member's bill has Valley people, which of the following been passed in the Parliament of India statements is/are correct ? for the first time in its history. 1) Rigvedic Aryans used the coat of mail Which of the statements given above is/are and helmet in warfare whereas the correct? people of Indus Valley Civilization did (a) 1 only not leave any evidence of using them 2) Rigvedic Aryans knew gold, silver and (b) 2 only copper whereas Indus Valley people (c) Both 1 and 2 knew only copper and iron. (d) Neither 1 nor 2 3) Rigvedic Aryans had domesticated the Explanation: horse whereas there is no evidence of Any MP who is not a minister is a private Indus Valley people having been aware member and he or she can submit a of this animal. legislative proposal for enacting it as law.. Select the correct answer using the codes Out of 14 private members’ bills enacted so given below: far since the commencement of Parliament (a) 1 only in 1952, five were introduced in the Rajya (b) 2 and 3 only Sabha and became law of the land. (c) 1 and 3 only The Rights of Transgender Persons Bill, (d) 1, 2 and 3 2014, passed by the Rajya Sabha in 2015 is Explanation: the first private member's bill to get the Statement 1 is correct upper house's approval in the past 45 years.  Rig Vedic Aryans used the coat of mail Members of parliament other than and helmet ministers are called private members and  These elements were not a feature of bills presented by them are known as the IVC private member's bills. Statement 2 is wrong The last Private Member's Bill passed by  While Rig Vedic Aryans knew gold, the parliament was the Supreme Court Harappans knew and used silver (Enlargement of Criminal Appellate Statement 3 is wrong Jurisdiction) Bill, 1968, which became an  The Rig Vedic Aryans have domesticated act on August 9, 1970 horses. In Lok Sabha, the last two and a half hours  The IVC people while they did not of a sitting on every Friday are generally domesticate horses, they were aware of allotted for transaction of “Private Members’ the existence of horses as their remains have been found in various sites such as: Business”, i.e., Private Members’ Bills and  Mohenjo-Daro, Harappa ("small Private Members’ Resolutions. Every horse"), member of Parliament, who is not a Minister,  Lothal (e.g., a terracotta figurine and is called a Private Member. a molar horse tooth, dated to 2200 Source: BC), http://www.business-  Surkotada (dated to 2400-1700 BC) standard.com/article/news-ians/only-14-

UPSC 2017 Preliminary Exam Q&A Video Discussion with Smart Leaders IAS Page 31 of 39 will be available in our youtube channel “Smart Leaders IAS Online” at 9:00pm

78. 'Recognition of Prior Learning Scheme' is It is a genetic link between the four sometimes mentioned in the news with other protected areas which it adjoins: reference to  Billigiriranga Swamy Temple Wildlife (a) Certifying the skills acquired by Sanctuary, construction workers through  Sigur Plateau,  Mudumalai National Park traditional channels.  Bandipur National Park (b) Enrolling the persons in Universities for distance learning programmes. 80. One of the implications of equality in (c) Reserving some skilled jobs to rural and society is the absence of urban poor in some public sector (a) Privileges undertakings. (b) Restraints (d) Certifying the skills acquired by trainees (c) Competition (d) Ideology under the National Skill Development Explanation: equality doesn't mean Programme. uniformity, but rather the absence of privilege based on power over others Explanation: Source: It is a scheme to certify the skills acquired http://www.bmartin.cc/pubs/08sa.html by workers in the unorganised sectors through traditional, non-formal learning 81. Consider the following statements in channels. respect of Trade Related Analysis of Fauna Source: and Flora in Commerce (TRAFFIC) :

http://indianexpress.com/article/india/ind 1) TRAFFIC is a bureau under United Nations Environment Programme ia-others/formal-badge-for-non-formal- (UNEP). skill/ 2) The mission of TRAFFIC is to ensure that trade in wild plants and animals is 79. From the ecological point of view, which not a threat to the conservation of one of the following assumes importance in nature. being a good link between the Eastern. Which of the given above statement(s) Ghats and the Western Ghats ? is/are correct? (a) Sathyamangalam Tiger Reserve (a) 1 only (b) Nallarnala Forest (b) 2 only (c) Nagarhole National Park (c) Both 1 and 2 (d) Seshachalam Biosphere Reserve (d) Neither 1 nor 2 Explanation: Explanation:  Sathyamangalam Wildlife Sanctuary and TRAFFIC was established in 1976. Tiger Reserve is a protected area and TRAFFIC is governed by the TRAFFIC tiger reserve along the Western Ghats in Committee, a steering group composed of the Indian state of Tamil Nadu members of TRAFFIC’s partner  It is the largest wildlife sanctuary in organizations, WWF and IUCN. Tamil Nadu TRAFFIC, the wildlife trade monitoring  Sathyamangalam forest range is a network, works to ensure that trade in wild significant wildlife corridor in the Nilgiri plants and animals is not a threat to the Biosphere Reserve between the conservation of nature. Western Ghats and the rest of the Source: Eastern Ghats http://www.traffic.org/overview/

UPSC 2017 Preliminary Exam Q&A Video Discussion with Smart Leaders IAS Page 32 of 39 will be available in our youtube channel “Smart Leaders IAS Online” at 9:00pm

82. Which principle among the following was 84. Which of the following gives 'Global Gender added to the Directive Principles of State Gap Index' ranking to the countries of the Policy by the 42nd Amendment, to the world? Constitution ? (a) World Economic Forum (a) Equal pay' for equal work for both men (b) UN Human Rights Council and women (c) UN Women (b) Participation of workers in the (d) World Health Organization management of industries Explanation: (c) Right to work, education and public Through the Global Gender Gap Report, the assistance World Economic Forum quantifies the (d) Securing living wage and human magnitude of gender disparities and tracks conditions of work to workers their progress over time, with a specific focus Explanation: on the relative gaps between women and Article 43A {Participation of workers in men across four key areas: health, education, economy and politics. management of industries} inserted by 42 Source: Amendment Act 1976 http://reports.weforum.org/global-gender- Source: gap-report-2016/ http://indiacode.nic.in/coiweb/amend/am end42.htm 85. Which of the following statements is/are correct regarding Smart India Hackathon 83. Which one of the following statements is 2017? correct? 1) It is a centrally sponsored scheme for (a) Rights are claims of the State against the developing every city of our country citizens. into Smart Cities in a decade. (b) Rights are privileges which are 2) It is an initiative to identify new digital incorporated in the Constitution of a technology innovations for solving the State. many problems faced by our country. (c) Rights are claims of the citizens 3) It is a programme aimed at making all against the State. the financial transactions in our country (d) Rights are privileges of a few citizens completely digital in a decade. against the many. Select the correct answer using the code Explanation: given below: “The Fundamental Rights are meant for (a) 1 and 3 only promoting the ideal of political democracy. (b) 2 only They prevent the establishment of an (c) 3 only authoritarian and despotic rule in the (d) 2 and 3 only country, and protect the liberties and Explanation: freedoms of the people against the invasion About 10,000 students from across the by the State. They operate as limitations on country are set to produce digital solutions the tyranny of the executive and arbitrary for problems faced by people in India. laws of the legislature.” Source: Source: http://www.hindustantimes.com/india- Indian Polity – Laxmikanth news/smart-india-hackathon-2017-10- 000-students-to-develop-digital- solutions/story- vvHrb0wqlYJFQ6VkutQhMI.html

UPSC 2017 Preliminary Exam Q&A Video Discussion with Smart Leaders IAS Page 33 of 39 will be available in our youtube channel “Smart Leaders IAS Online” at 9:00pm

86. Which of the following statements is/are (b) 1 and 3 only correct regarding the Monetary Policy (c) 2 and 3 only Committee (MPC)? (d) 1 only 1) It decides the RBI's benchmark interest Explanation: rates. Statement 1 is correct 2) It is a 12-member body including the Governor of RBI and is reconstituted  Manipuri Sankirtana is a form of every year. performing art involving ritual singing, 3) It functions under the chairmanship of drumming and dancing performed in the Union Finance Minister. the temples and domestic spaces in Select the correct answer using the code Manipur given below: Statement 2 is wrong (a) 1 only  Not just cymbals, but drums are also (b) 1 and 2 only used as mentioned earlier (c) 3 only (d) 1, 2 and 3 Statement 3 is correct Explanation:  The performers narrate the many The Monetary Policy Committee would be stories of . It is practiced entrusted with the task of fixing the primarily by the Vaishnava community benchmark policy rate (repo rate) required to contain inflation within the 88. Who among the following was/were specified target level. As per the provisions associated with the introduction of of the RBI Act, out of the six Members of Ryotwari Settlement in India during the Monetary Policy Committee, three Members will be from the RBI and the other three British rule? Members of MPC will be appointed by the 1) Lord Cornwallis Central Government. 2) Alexander Read The Governor of the Bank—Chairperson, ex 3) Thomas Munro officio; Select the correct answer using the codes Source: given below: http://pib.nic.in/newsite/PrintRelease.asp (a) 1 only x?relid=151264 (b) 1 and 3 only

87. With reference to Manipuri Sankirtana (c) 2 and 3 only consider the following statements (d) 1, 2 and 3 1) It is a song and dance performance. Explanation: 2) Cymbals are the only musical  Ryotwari system was one of the three instruments used in the performance. principal methods of revenue collection 3) It is performed to narrate the life and in British India. deeds of Lord Krishna.  The system was devised by Capt. Which of the statements given above is/are Alexander Read and Thomas (later Sir correct? Thomas) Munro at the end of the 18th (a) 1, 2 and 3 century and introduced by the latter

UPSC 2017 Preliminary Exam Q&A Video Discussion with Smart Leaders IAS Page 34 of 39 will be available in our youtube channel “Smart Leaders IAS Online” at 9:00pm

when he was governor (1820–27) of and lead are not readily absorbed or Madras captured by microorganisms. A recent  It was prevalent in most of southern experiment, however, suggests that fish India, being the standard system of the bones have some success absorbing lead Madras Presidency from contaminated soil. Bone char has been  The Permanent Settlement was shown to bioremediate small amounts of concluded in 1793 by the Company cadmium, copper, and zinc. administration headed by Charles, Earl Source: Cornwallis, also known as Lord https://en.wikipedia.org/wiki/Bioremediat Cornwallis ion

89. In the context of solving pollution problems, 90. The Trade Disputes Act of 1929.provided what is/are the advantage/advantages of for bioremediation technique? (a) the participation of workers in the 1) It is a technique for cleaning up management of industries. pollution by enhancing the same (b) arbitrary powers to the management to biodegradation process that occurs in quell industrial disputes. nature. (c) an intervention by the British Court in 2) Any contaminant with heavy metals the event of a trade dispute. such as cadmium and lead can be (d) a system of tribunals and a ban on readily and completely treated by strikes. bioremediation using microorganisms. Explanation: 3) Genetic engineering can be used to  This was a British Act of Parliament create microorganisms specifically passed in response to the General Strike designed for bioremediation. of 1926 in the UK, introduced by the Select the correct answer using the codes Attorney General for England and given below: Wales, Sir Douglas Hogg MP (a) 1 only  The Act declared unlawful secondary (b) 2 and 3 only action and any strike whose purpose (c) 1 and 3 only was to coerce the government of the day (d) 1, 2 and 3 directly or indirectly Explanation:  An unlawful strike was made a criminal Bioremediation processes typically involve offence, punishable by imprisonment for the actions of many different microbes up to two year acting in parallel or sequence to complete the degradation process. 91. Local self-government can be best A widely used approach to bioremediation explained as an exercise in involves stimulating naturally occurring (a) Federalism microbial communities, providing them (b) Democratic decentralization with nutrients and other needs, to break (c) Administrative delegation down a contaminant. (d) Direct democracy Not all contaminants are easily treated by Explanation: bioremediation using microorganisms. For In this report on Local Governance, the example, heavy metals such as cadmium Administrative Reforms Commission has UPSC 2017 Preliminary Exam Q&A Video Discussion with Smart Leaders IAS Page 35 of 39 will be available in our youtube channel “Smart Leaders IAS Online” at 9:00pm

examined in detail the issues relating to Explanation: rural and urban local governance in India A Digital Single Market (DSM) is one in with a special focus on the need for real which the free movement of persons, democratic decentralisation in the country services and capital is ensured and where in order to usher in genuine grass roots the individuals and businesses can democracy as envisaged by the founding seamlessly access and exercise online fathers of our republic and as now activities under conditions of fair specifically mandated by our Constitution. competition, and a high level of consumer Source: and personal data protection, irrespective http://arc.gov.in/6-1.pdf of their nationality or place of residence. On 10 May 2017 the European Commission 92. Consider the following statements: published the mid-term review of the With reference to the Constitution of India, Digital Single Market Strategy. It shows the the Directive Principles of State Policy progress made in implementing the constitute limitations upon Strategy since 2015 and where further 1) legislative function. actions are needed. 2) executive function. The European Commission has identified Which of the given above statement(s) the completion of the Digital Single Market is/are correct? (DSM) as one of its 10 political priorities. (a) 1 only Source: (b) 2 only (c) Both 1 and 2 94. At one of the places in India, if you stand on (d) Neither 1 nor 2 the seashore and watch the sea, you will For this question, there lies some find that the sea water recedes from the ambiguity. According to Article 37 of the shore line a few kilometres and comes back Indian Constitution, it prescribes DPSP as to the shore, twice a day, and you can fundamental to the governance of the actually walk on the sea floor when the country. Therefore, it limits both legislative water recedes. This unique phenomenon is and executive arms of the tate. seen at At the same time, DPSP is not enforceable (a) Bhavnagar and considered as instrument of (b) Bheemunipatnam instructions. Therefor it attains a positive (c) Chandipur tone which requests State to realise these (d) Nagapattinam principles. In this aspect, it is not to be Explanation: considered as limitation rather in the form  Chandipur also known as Chandipur-on- of positive rapproachment for its sea is a small sea resort in Odisha implementation.  The beach is unique in that the water recedes up to 5 km during the ebb tide. 93. The term 'Digital Single Market Strategy'  Ebb tide means the period between high seen in the news refers to tide and low tide during which water (a) ASEAN flows away from the shore (b) BRICS  Due to its unique circumstances, the (c) EU beach supports bio-diversity (d) G20

UPSC 2017 Preliminary Exam Q&A Video Discussion with Smart Leaders IAS Page 36 of 39 will be available in our youtube channel “Smart Leaders IAS Online” at 9:00pm

95. With reference to the 'Prohibition of Benami Property Transactions Act, 1988 96. Due to some reasons, if there is a huge fall (PBPT Act)', consider the following statements in the population of species of butterflies, 1) A property transaction is not treated as what could be its likely a benami transaction if the owner of the consequence/consequences? property is not aware of the transaction. 1) Pollination of some plants could be 2) Properties held benami are liable for adversely affected. confiscation by the Government. 2) There could be a drastic increase in the 3) The Act provides for three authorities fungal infections of some cultivated for investigations but -does not provide plants. for any appellate mechanism. 3) It could lead to a fall in the population of Which of the statements given above is/are some species of wasps, spiders and correct? birds. (a) 1 only Select the correct answer using the code (b) 2 only given below: (c) 1 and 3 only (a) 1 only (d) 2 and 3 only (b) 2 and 3 only Section 5(1). Property held benami liable to (c) 1 and 3 only acquisition. All properties held benami shall be subject to acquisition by such authority, (d) 1, 2 and 3 in such manner and after following such Explanation: procedure, as may be prescribed. Butterflies are an important pollination. As per Sec 2 which defines the various Hence if they disappear, it will affect the terms included in the Act, clause (b) says plant’s reproductive functions. It makes the that a transaction or an arrangement in statement (1) correct. respect of a property where the owner of As such there is no link between fungal infections of plants and butterflies. Thus, the property is not aware of, or, denies when butterflies disappear, it may not have knowledge of, such ownership will be a considerable impact on fungal infection. considered as Benami transaction. Therefore, the statement (2) is wrong. Section 23 of the Act says that the Initiating Some of the common predators of butterflies include but are certainly not Officer, after obtaining prior approval of the limited to: wasps, ants, parasitic flies, birds, Approving Authority, shall have power to snakes, toads, rats, lizards, dragonflies and conduct or cause to be conducted any even monkeys. Hence, when the butterflies inquiry or investigation in respect of any are lost, it will have an impact on species person, place, property, assets, documents, that depend on it. This makes the statement (3) correct. books of account or other documents, in Source: respect of any other relevant matters under http://archive.naplesnews.com/lifestyle/h this Act ome/wings-at-work-butterflies-pollinate- Source: plants-but-in-ways-different-from-all- http://www.incometaxindia.gov.in/page others-ep-394968174-339851142.html http://www.thebutterflysite.com/what- s/acts/prohibition-of-benami-property- eats-butterflies.shtml transactions-act-1988.aspx

UPSC 2017 Preliminary Exam Q&A Video Discussion with Smart Leaders IAS Page 37 of 39 will be available in our youtube channel “Smart Leaders IAS Online” at 9:00pm

97. It is possible to produce algae based of cotton or wheat, but less than that of biofuels, but what is/are the likely corn, to replenish the water lost in limitation(s) of developing countries in evaporation. Water will remain a central promoting this industry issue for algae biofuels production and will 1) Production of algae based biofuels is possible in seas only and not on need to be considered carefully as the continents. industry expands. 2) Setting up and engineering the algae Photosynthetic efficiency in string sunlight based biofuel production requires high falls short of the theoretical potential level of expertise/technology until the resulting in low yields which are the major construction is completed. culprits for the forbiddingly high 3) Economically viable production production cost of algal cell mass. necessitates the setting up of large scale Source: facilities which may raise ecological and https://en.wikipedia.org/wiki/Algae_fuel social concerns. https://www.ncbi.nlm.nih.gov/pmc/article Select the correct answer using the code s/PMC3152439/ given below (a) 1 and 2 only 98. Which of the following are the objectives of (b) 2 and 3 only `National Nutrition Mission'? (c) 3 only 1) To create awareness relating to (d) 1, 2 and 3 malnutrition among pregnant women Explanation: and lactating mothers. There are two main methods of algae 2) To reduce the incidence of anaemia cultivation – ponds and Photobioreactors. among young children, adolescent girls Ponds can be based both in continents and and women. seas. 3) To promote the consumption of millets, In brief, there are significant challenges for coarse cereals and unpolished rice. engineers to either design photobioreactors 4) To promote the consumption of poultry (PBRs) that are cheap enough for large- eggs. scale deployment, or for engineers and Select the correct answer using the codes biologists to combine forces to develop given below: species that grow efficiently in low-cost (a) 1 and 2 only open systems. (b) 1, 2 and 3 only Oil extraction is another challenge that is (c) 1, 2 and 4 only most easily addressed from the engineering (d) 3 and 4 only side. There are three major strategies for The key objectives of these programmes are extracting oil from algae: oil press/expeller, as under: hexane extraction, and supercritical CO2  To create awareness relating to fluid extraction. These technologies have all malnutrition amongst pregnant women, been successfully demonstrated but are lactating mothers, promote healthy relatively expensive, either in terms of lactating practices and importance of equipment needed or energy required to balanced nutrition; extract the oil.  To improve maternal and child under- Algae grown in open ponds have water nutrition in 200 high burdened districts requirements per unit area similar to that and to prevent and reduce the under-

UPSC 2017 Preliminary Exam Q&A Video Discussion with Smart Leaders IAS Page 38 of 39 will be available in our youtube channel “Smart Leaders IAS Online” at 9:00pm

nutrition prevalent among children 100. In the context of mitigating the below 3 years; impending global warming due to  To reduce incidence of anaemia among anthropogenic emissions of carbon dioxide, young children, adolescent girls and which of the following can be the potential women. sites for carbon sequestration ? http://pib.nic.in/newsite/PrintRelease.asp 1) Abandoned and uneconomic coal seams x?relid=103192 2) Depleted oil and gas reservoirs 3) Subterranean deep saline formations 99. Consider the following statements: Select the correct answer using the codes 1) The Factories Act, 1881 was passed with given below: a view to fix the wages of industrial (a) 1 and 2 only workers and to allow the workers to (b) 3 only form trade unions. (c) 1 and 3 only 2) N.M. Lokhande was a pioneer in (d) 1, 2 and 3 organizing the labour movement in Explanation: British India. In nature, coal seams also contain gases Which of the above statements is/are such as methane. If CO2 is injected into a correct? coal seam it displaces the methane, which (a) 1 only can then be recovered. The CO2 will remain (b) 2 only stored within the seam, providing the coal (c) Both 1 and 2 is never disturbed. (d) Neither 1 nor 2 Carbon dioxide storage in geologic Explanation: formations includes oil and gas reservoirs, Statement 1 is wrong unmineable coal seams, and deep saline  The factories act of 1881 dealt reservoirs. These are structures that have primarily with the problem of child stored crude oil, natural gas, brine and CO2 labour. Its provisions were: over millions of years.  Those children between the age of 7 In some cases, production from an oil or and 12 would not work for more natural gas reservoir can be enhanced by than 9 hours a day pumping CO2 into the reservoir to push out  Children would get four holidays per the product, which is called enhanced oil month recovery.  Proper fencing off of dangerous Storage of CO2 in deep saline formations machinery does not produce value-added by-products, Statement 2 is correct but it has other advantages. First, the  Narayan Meghaji Lokhande was a estimated carbon storage capacity of saline pioneer of the labour movement in formations in the United States is large, India. making them a viable long-term solution.  Lokhande is acclaimed as the Father Source: of the Trade Union Movement in https://energy.gov/fe/science- India innovation/carbon-capture-and-storage-  He is remembered not only for research/carbon-storage-rd ameliorating the working conditions of textile mill-hands in the 19th century but also for his courageous initiatives on caste and communal issues

UPSC 2017 Preliminary Exam Q&A Video Discussion with Smart Leaders IAS Page 39 of 39 will be available in our youtube channel “Smart Leaders IAS Online” at 9:00pm